1answer.
Ask question
Login Signup
Ask question
All categories
  • English
  • Mathematics
  • Social Studies
  • Business
  • History
  • Health
  • Geography
  • Biology
  • Physics
  • Chemistry
  • Computers and Technology
  • Arts
  • World Languages
  • Spanish
  • French
  • German
  • Advanced Placement (AP)
  • SAT
  • Medicine
  • Law
  • Engineering
Marrrta [24]
3 years ago
9

URGENT HELP NEEDED WITH TRIANGLES/CIRCLES! IF ET = 31, DE = 65, AND DC= 74, FIND THE PERIMETER OF DCE.

Mathematics
1 answer:
snow_tiger [21]3 years ago
6 0

ES = ET

and

DS = DR

and

CR = CT ( because each pair are tangents to the circle)


So ES = 31 and Ds = 65-31 = 34

and DR = 34 and CR = 74- 34 = 40.

CT = CR = 40

Therfore the line EC = 31 + 40 = 71


Finally the perimeter = 65 + 74 + 71 = 210 (answer)

You might be interested in
Consider two people being randomly selected. (For simplicity, ignore leap years.)
inna [77]

Answer:

(a) = \frac{144}{133225} \\\\(b) = \frac{1}{365}

Step-by-step explanation:

Part (a) the probability that two people have a birthday on the 9th of any month.

Neglecting leap year, there are 365 days in a year.

There are 12 possible 9th in months that make a year calendar.

If two people have birthday on 9th; P(1st person) and P(2nd person).

=\frac{12}{365} X\frac{12}{365}  = \frac{144}{133225}

Part (b) the probability that two people have a birthday on the same day of the same month

P(2 people selected have birthday on the same day of same month) + P(2 people selected not having birthday on  same day of same month) = 1

P(2 people selected not having birthday on  same day of same month):

= \frac{365}{365} X \frac{364}{365} =\frac{364}{365}

P(2 people selected have birthday on the same day of same month) = 1-\frac{364}{365} \\\\= \frac{1}{365}

7 0
3 years ago
Zappos is an online retailer based in Nevada and employs 1,300 employees. One of their competitors, Amazon, would like to test t
Amanda [17]

Answer:

t=\frac{33.9-36}{\frac{4.1}{\sqrt{22}}}=-2.402    

df = n-1= 22-1=21

t_{\alpha/2}= -2.08

Since the calculated values is lower than the critical value we have enough evidence to reject the null hypothesis at the significance level of 2.5% and we can say that the true mean is lower than 36 years old

Step-by-step explanation:

Data given

\bar X=33.9 represent the sample mean

s=4.1 represent the sample standard deviation

n=22 sample size  

\mu_o =26 represent the value that we want to test

\alpha=0.025 represent the significance level for the hypothesis test.  

t would represent the statistic (variable of interest)  

p_v represent the p value for the test (variable of interest)  

System of hypothesis

We need to conduct a hypothesis in order to check if the true mean is less than 36 years old, the system of hypothesis would be:  

Null hypothesis:\mu \geq 36  

Alternative hypothesis:\mu < 36  

The statistic is given by:

t=\frac{\bar X-\mu_o}{\frac{s}{\sqrt{n}}}  (1)  

And replacing we got:

t=\frac{33.9-36}{\frac{4.1}{\sqrt{22}}}=-2.402    

Now we can calculate the critical value but first we need to find the degreed of freedom:

df = n-1= 22-1=21

So we need to find a critical value in the t distribution with df =21 who accumulates 0.025 of the area in the left and we got:

t_{\alpha/2}= -2.08

Since the calculated values is lower than the critical value we have enough evidence to reject the null hypothesis at the significance level of 2.5% and we can say that the true mean is lower than 36 years old

3 0
3 years ago
PLS HELP ASAP! GIVING BRAINLY
ioda
The answer would be (x+1) and (x+10) so E
4 0
2 years ago
Read 2 more answers
Nvm no one helped me, answer is 17/25 -.-
geniusboy [140]
The answer is 17/25. Hope this helps! :) I know how you feel! 
3 0
3 years ago
5,049 +48=105
Alborosie
<h2>Solution (a) :-</h2>

Let the cost of the less expensive comic book be x .

Then :-

1284 - 11x = x

Using this equation , we can find the cost of the less expensive comic book .

Price of the less expensive book :-

1284 - 11x = x

11x + x = 1284

12x = 1284

x =  \frac{1284}{12}

x = 107

Cost of the less expensive book is 107 .

Therefore , the epice of the less expensive book = $ 107 .

<h2>Solution (b) :-</h2>

Cost of the more expensive book = 11x

Which means :-

1284 - x = 11x

11x + x = 1284

12x = 1284

x = 107

11x = 11 \times 107

= 1177

Therefore , the price of the more expensive book = $ 1177 .

4 0
3 years ago
Other questions:
  • I still cant figure it out . ughh −2∣5w−7∣+9=−7
    7·2 answers
  • What property can be used to solve this equation?
    12·2 answers
  • Plz show me how to work this out
    12·1 answer
  • 1/2x = 17.31 I need help finding out what can equals??
    11·2 answers
  • A bag contains 2 RED beads, 7 BLUE beads, and 6 GREEN beads. If a single bead is picked at random, what is the probability that
    12·2 answers
  • I need help with this​
    8·1 answer
  • Help me graph this pls!<br> y=-(2)/(3)x+2
    7·1 answer
  • Andrew is two years older than Beatrice, and Chris is three years younger than Beatrice.
    13·1 answer
  • BRAINLIEST AND EXTRA POINTS IF U HELP !
    9·2 answers
  • If Angle 4 = 6x+8 and Angle 3 = 20x+16 find the measure of all Four Angles
    13·1 answer
Add answer
Login
Not registered? Fast signup
Signup
Login Signup
Ask question!